매개 변수가있는 두 가지 부등식 $a,b,c>0$ 그런 $ca+ab+bc+abc\leq 4$
허락하다 $a,b,c>0$ 그렇게 $bc+ca+ab+abc\leq 4$. 다음과 같은 불평등을 증명하십시오.
(ㅏ) $8(a^2+b^2+c^2)\geq 3(b+c)(c+a)(a+b)$, 및
(비) $\dfrac{1}{a^2}+\dfrac{1}{b^2}+\dfrac{1}{c^2}+\dfrac{2}{a^2b}+\dfrac{2}{b^2c}+\dfrac{2}{c^2a}\geq 9$.
두 불평등에 대한 고유 한 평등 사례가 다음과 같이 주어진다는 것도 증명하십시오. $a=b=c=1$.
다음은 유용하거나 관련있는 결과입니다.
- https://artofproblemsolving.com/community/c6h1241430p6342224
- https://artofproblemsolving.com/community/c6h284290p1535893
- https://artofproblemsolving.com/community/c6h608971p3619202
- https://artofproblemsolving.com/community/c6h1804479p11995588
- 만약 $ab+bc+ca+abc=4$, 다음 $\sqrt{ab}+\sqrt{bc}+\sqrt{ca}\leq 3\leq a+b+c$
이러한 링크의 불평등을 해결하는 데 사용되는 기술은 불평등을 증명하는 데 유용 할 수 있습니다.
시도. 가장 간단한 경우$a=b=c=:t$, 우리는 $t^3+3t^2-4\leq 0$, 언제 $0<t\leq 1$. 따라서 불평등 (a)와 (b)는$$24t^2\geq 24t^3$$ 과 $$\frac{3}{t^2}+\frac{6}{t^3}\geq 9\,,$$분명히 사실입니다. 이러한 불평등을 일반적으로 증명하는 방법은 무엇입니까?
답변
첫 번째 불평등.
허락하다 $a=kx$, $b=ky$ 과 $c=kz$ 그런 $k>0$ 과 $xy+xz+yz+xyz=4.$
따라서 조건은 $$k^2(xy+xz+yz)+k^3xyz\leq xy+xz+yz+xyz$$ 또는 $$(k-1)((k+1)(xy+xz+yz)+(k^2+k+1)xyz)\leq0$$ 또는 $$k\leq1.$$ 따라서 우리는 $$8(x^2+y^2+z^2)\geq3k(x+y)(x+z)(y+z)$$ 이후 $0<k\leq1$, 증명하는 것으로 충분합니다. $$8(x^2+y^2+z^2)\geq3(x+y)(x+z)(y+z).$$ 이제 다음 형식으로 새 조건을 다시 작성하십시오. $$\sum_{cyc}\frac{1}{x+2}=4$$ 그리고하자 $x=\frac{2p}{q+r}$ 과 $y=\frac{2q}{p+r},$ 어디 $p$, $q$ 과 $r$ 긍정적입니다.
그러므로, $z=\frac{2r}{p+q}$ 그리고이 대체 후에 우리는 명백한 것을 얻습니다.
하지만 그 전에 증명하는 것이 낫습니다 $$x+y+z\geq xy+xz+yz,$$ 이를 위해 다음 사항을 증명해야합니다. $$\sum_{cyc}\frac{2p}{q+r}\geq\sum_{cyc}\frac{4pq}{(p+r)(q+r)}$$ 또는 $$\sum_{cyc}p(p+q)(p+r)\geq2\sum_{cyc}pq(p+q)$$ 또는 $$\sum_{cyc}(p^3-p^2q-p^2r+pqr)\geq0,$$ 그것은 Schur에 의해 사실입니다.
이제부터 $$1\geq\frac{xy+xz+yz}{x+y+z},$$ 증명하는 것으로 충분합니다 $$8(x^2+y^2+z^2)(xy+xz+yz)\geq3(x+y+z)(x+y)(x+z)(y+z)$$ 또는 $$\sum_{cyc}(5x^3y+5x^3z-6x^2y^2-4x^2yz)\geq0,$$ 뮤어 헤드도 마찬가지입니다.
이것은 라그랑주 승수를 사용하는 접근 방식입니다.
첫 번째 부분에서는 문제를 다음과 같이 작성할 수 있습니다. \begin{align}\min&\quad8(a^2+b^2+c^2)-3(a+b)(a+c)(b+c)\\\text{s.t.}&\quad ab+ac+bc+abc=4-\epsilon\\&\quad a,b,c>0\quad\land\quad0\le\epsilon<4.\end{align} 그런 다음 우리는 $\mathcal L=f-\lambda g$ 어디 $f(a,b,c)=8(a^2+b^2+c^2)-3(a+b)(a+c)(b+c)$ 과 $g(a,b,c)=ab+ac+bc+abc-(4-\epsilon)$. 편미분은 다음과 같습니다.\begin{align}\mathcal L_a&=16a-3(b+c)(2a+b+c)-\lambda(b+c+bc)\\\mathcal L_b&=16b-3(a+c)(2b+a+c)-\lambda(a+c+ac)\\\mathcal L_c&=16c-3(a+b)(2c+a+b)-\lambda(a+b+ab)\\\mathcal L_\lambda&=ab+ac+bc+abc-(4-\epsilon).\end{align} 다음으로 \begin{align}\mathcal L_a-\mathcal L_b&=0\implies16+3(a+b)+\lambda(1+c)=0,a=b\\\mathcal L_a-\mathcal L_c&=0\implies16+3(a+c)+\lambda(1+b)=0,a=c\\\mathcal L_b-\mathcal L_c&=0\implies16+3(b+c)+\lambda(1+a)=0,b=c,\end{align} 그래서 일반성을 잃지 않고 우리는 $a=b$. 시키는$c=ka$ 수확량 $f(a,b,c)=8(2+k^2)a^2-6(1+k)^2a^3$ 과 $g(a,b,c)=(1+2k)a^2+ka^3-(4-\epsilon)$. 이후$f\to0^+$ 같이 $\epsilon\to4^-$ 우리는 찾는 것을 목표로한다 $k,\epsilon$ 그런 $f\le0$.
초등 미적분은 $f(a;k)$ 간격에서 단조롭게 증가 $[0,k^*]$ 어디 $k^*=8(2+k^2)/(9(1+k)^2)$, 보낸 사람 $0$ ...에 $f(k^*;k)>0$. 에 대한$a>k^*$, 함수 $f(a;k)$ 단조롭게 감소 $-\infty$, 여기서 축을 만나는 $a=3k^*/2$. 긍정적 인 루트$g$ 때 가장 크다 $4-\epsilon$가장 크다. 그건,$\epsilon=0$. 이 값에서는$$g\left(\frac{3k^*}2;k\right)=(1+2k)\left(\frac{4(2+k^2)}{3(1+k)^2}\right)^2+k\left(\frac{4(2+k^2)}{3(1+k)^2}\right)^3-4=(k-1)^2\cdot\frac{P(k)}{Q(k)}$$ 어디 $P,Q$계수가 양의 다항식입니다. 같이$k>0$ 유일한 해결책 $g=0$ 이다 $k=1$, 그 뒤에 $a=b=c=1$. $\square$
문제의 원인은 다음과 같습니다. https://math.stackexchange.com/questions/2825783/problems-regarding-inequality. 아래는 두 부분에 대한 솔루션이며 일부는 위의 이전 스레드에서 다른 사용자가 작성했습니다.
파트 (a) : 진실에 의한 해결책 .
허락하다 $a = \dfrac{2kx}{y+z}$, $b = \dfrac{2ky}{z+x}$, 및 $c = \dfrac{2kz}{x+y}$, 어디 $x,y,z,k>0$. 주어진 조건에서 우리는$k \leqslant 1$ 필요한 불평등은 다음과 같습니다. $$96k^2\,\sum_\text{cyc}\, \frac{x^2}{(y+z)^2} \geqslant 72k^3 \prod_\text{cyc}\, \left(\frac{x}{y+z}+\frac{y}{z+x}\right)\,.$$ 이것은 $$4\,\sum_\text{cyc}\, \frac{x^2}{(y+z)^2} \geqslant 3k\,\prod_\text{cyc}\, \left(\frac{x}{y+z}+\frac{y}{z+x}\right)\,.$$
때문에 $ k \leqslant 1$, 그것을 보여주는 것으로 충분합니다 $$4\,\sum_\text{cyc}\, \frac{x^2}{(y+z)^2} \geqslant 3\, \prod_\text{cyc}\, \left(\frac{x}{y+z}+\frac{y}{z+x}\right)\,,$$ 그러나 마지막 부등식은 다음과 같습니다. $$\sum_\text{cyc}\, \frac{\big(2x^4+16xyz^2+6y^2z^2+(2x^2+9xz+9yz+6z^2)(x+y-z)^2\big)(x-y)^2}{(x+y)^2(y+z)^2(z+x)^2} \geqslant 0\,.$$
Part (b) : 솔루션 by Michael Rozenberg . (@Michael,이 증명을 별도의 답변에 사용하려면 여기 내 답변에서이 솔루션을 제거하고 자신의 답변에 추가 할 수 있습니다.)
AM-GM 불평등으로 $$4\geq3\sqrt[3]{a^2b^2c^2}+abc\,,$$ 주는 $$abc\leq1\,.$$ 따라서 AM-GM 불평등에 의해 우리는 $$\sum_\text{cyc}\,\frac{1}{a^2}+2\,\sum_\text{cyc}\,\frac{1}{a^2b}\geq\frac{3}{\sqrt[3]{a^2b^2c^2}}+\frac{6}{abc}\geq9\,.$$
파트 (b) : 나에 의한 대체 증명.
제약 불평등과 함께 AM-GM 불평등을 사용하면 $$4\geq bc+ca+ab+abc\geq 4\,\sqrt[4]{(bc)(ca)(ab)(abc)}=4\,(abc)^{\frac{3}{4}}\,,$$ 어떻게 $$abc\leq 1\,.$$ 자, 우리는 AM-GM 불평등에 의해 $$\sum_\text{cyc}\,\frac{1}{a^2}+2\,\sum_\text{cyc}\,\frac{1}{a^2b}\geq 9\, \sqrt[9]{\left(\prod_\text{cyc}\,\frac{1}{a^2}\right)\,\left(\prod_\text{cyc}\,\frac{1}{a^2b}\right)^2}=\frac{9}{(abc)^{\frac{8}{9}}}\,.$$ 같이 $abc\leq 1$, 필요한 부등식이 즉시 이어집니다.